Plastic and Reconstructive Surgery Flashcards

1
Q
  1. All of the following are true about split-thickness skin grafts EXCEPT

A. Degree of contraction is dependent on amount of dermis in graft.

B. High reliability of take.

C. Healing with abnormal pigmentation more common
in thin than thick grafts.

D. Meshing grafts improve their ultimate cosmetic
appearance.

A

D. Meshing grafts improve their ultimate cosmetic
appearance.

Many of the characteristics of a split-thickness graft are determined by the amount of dermis present.

Less dermis translates into less primary contraction (the degree to which a graft shrinks in surface area after harvesting and before grafting), more secondary contraction (the degree to which a graft shrinks during healing), and better chance of graft survival.

Thin split grafts have low primary contraction, high secondary contraction, and high reliability of graft take, often even in imperfect recipient beds.

Thin grafts, however, tend to heal with abnormal pigmentation and poor durability compared with thick split grafts and full-thickness grafts.

Split grafts may be meshed to expand the surface area that can be covered. (See Schwartz 10th ed., p. 1832.)

How well did you know this?
1
Not at all
2
3
4
5
Perfectly
2
Q

Which of the following definitions is INCORRECT?

A. Flap composition: Description o the tissue components within the flap.

B. Flap contiguity: The position of a flap relative to its
recipient bed.

C. Pedicle: Bridge of tissue that remains between a flap and its source; blood vessels that nourish a flap.

D. Free flap: Flaps that are completely detached rom the body prior to their reimplantation with microvascular anastomoses.

A

B. Flap contiguity: The position of a flap relative to its
recipient bed.

The composition of a flap describes its tissue components.
The contiguity of a flap describes its position related to its source.

Distant flaps are transferred from a different anatomic region to the defect.

They may remain attached to the source anatomic region (pedicled flaps) or may be transferred as free flaps by microsurgery.

These are completely detached from the body, and their blood supply is reinstated by microvascular anastomoses to recipient vessels close to the defect.

The term pedicle was originally used to describe a bridge of tissue that remains between a flap and its source, similar to how a peninsula remains attached to its mainland.

However, as knowledge of flap blood supply and (micro)vascular anatomy has improved over the years, the term pedicle has increasingly become reserved for describing the blood vessels that nourish the flap.(SeeSchwartz10thed.,p.1833.)

How well did you know this?
1
Not at all
2
3
4
5
Perfectly
3
Q

Factors infuencing the development of cleft lip/palate include all the following EXCEPT:

A. Increased parental age.

B. Vitamin A deficiency during pregnancy.

C. Infections during pregnancy.

D. Smoking during pregnancy.

A

Answer: B

The cause of orofacial clefting is felt to be multifactorial. Factors that likely increase the incidence of clefting include increased parental age, drug use and infections during pregnancy, and smoking during pregnancy. (See Schwartz 10th ed., pp. 1840–1841.)

How well did you know this?
1
Not at all
2
3
4
5
Perfectly
4
Q

Principles of reconstructive surgery include all of the following EXCEPT

A. Adequate restoration of lost anatomic components without residual deficits.

B. Uncomplicated and timely wound healing.

C. Individualization of specific reconstructive technique to specific patient deficit.

D. Compromise of extent of tumor resection if needed or specific reconstructive surgical outcome.

A

D. Compromise of extent of tumor resection if needed for specific reconstructive surgical outcome.

The reconstructive surgeon aims to restore lost anatomic components adequately.

Residual deficits, seemingly inconsequential, may progress to psychological morbidity, societal malacceptance, and social withdrawal.

Uncomplicated and timely wound healing is important to allow adjuvant therapies when indicated and smooth discharge to home and occupation.

Each defect can be addressed by a number of methods, but the technique must be decided or each individual patient.

Although a more complex reconstruction might offer improved outcomes, it may bring an increased risk of complications.

Some patients may therefore benefit from use of a simpler method with more acceptable anesthetic and operative risk rather than a gold-standard reconstruction. (See Schwartz 10th ed., p. 1862.)

How well did you know this?
1
Not at all
2
3
4
5
Perfectly
5
Q

Which of the following is false regarding the blood supply of skin grafts?

A. Full-thickness skin grafts (FTSGs) revascularize faster because they are transferred with their own blood supply.

B. For the first 48 h, a skin graft derives nutrients from passive diffusion from capillaries in the recipient bed.

C. Inosculation is revascularization of the graft through the formation of connections between vessels in the graft and vessels in the wound bed.

D. Arterial blood flow in the new graft is established before venous flow.

E. Mature circulation is established by days 5 to 7.

A

A. Full-thickness skin grafts (FTSGs) revascularize faster because they are transferred with their own blood supply.

COMMENTS: A. False.

Split-thickness skin grafts (STSGs) are composed of the epidermis and a portion of the dermis, whereas FTSGs include the epidermis and the entire dermis.

An FTSG is not transferred with its own blood supply, and both STSGs and FTSGs undergo the same steps of revascularization.

To successfully achieve good blood flow, the graft must be placed in a well- vascularized wound bed.

Vascularization takes longer in an FTSG because there is more tissue to be traversed for vascular connections to be established and a more robust vascular supply is needed to support the higher volume of tissue.

Similarly, a thinner STSG revascularizes more quickly than a thicker STSG.

B. True. During the first 48 h, serum diffuses into the graft from capillaries in the wound bed, a process called plasmatic imbibition.

C. True. Revascularization occurs after 48 h through a process called inosculation where vessels from the wound bed align with vessels within the graft. These connections mature with time but are initially tenuous and prone to blood pooling and pendulum-like flow. Neovascularization, or the growth of new blood vessels from the wound bed into the graft, also takes place.

D. True. The arterial vascularization happens faster than venous vascularization. The practice of elevating a newly grafted extremity to reduce venous congestion is based on this delayed development of venous outflow.

E. True. By days 5 to 7, the vasculature is mature and has established proper afferent and efferent flow. A fibrin layer initially forms, which holds the graft in place; by day 7, it is replaced by fibroblasts; and by days 10 to 14, the graft is firmly adherent.

The donor site is typically treated with a moist occlusive dressing for promotion of epithelialization or with primary closure in the case of FTSGs.

A split-thickness donor site maintains dermal appendages that provide stem cells for reepithelialization of the donor site.

The donor site for an FTSG must be primarily closed as the dermal appendages are removed along with the graft.

Skin grafts are prone to shearing forces and failure secondary to hematoma or seroma.

A bolster dressing or splint must be applied to immobilize the graft to prevent shearing of the immature vasculature and to provide compression to prevent the formation of seroma or hematoma that lifts the graft off the wound bed and interferes with revascularization.

Sensation returns to the graft over time, with reinnervation beginning at 4 to 5 weeks and completed by 1 to 2 years.

Sensation does not return to that of normal skin, but protective sensation is typically attained. Pain returns first, with light touch and tempera- ture following.

How well did you know this?
1
Not at all
2
3
4
5
Perfectly
6
Q
  1. Which of the following is not an advantage of FTSGs over STSGs?

A. Less secondary contracture at the recipient site

B. Ability to cover greater surface area

C. Maintains texture and appearance of normal skin

D. More durability when subject to trauma

E. More mobility at graft site, allowing for use over a joint

A

ANSWER: B

COMMENTS: A. Advantage. STSGs are harvested by a derma- tome, a machine that shaves off the epidermis and a variable portion of dermis at a set width and depth. Split-thickness grafts undergo more secondary contracture compared with STSGs. (In contrast, FTSGs undergo more primary contracture, while STSGs have less primary contracture allowing them to cover larger areas.)

B. Not Advantage for FTSG. STSGs can be meshed to increase the width and allow for additional coverage over a larger surface area. Additionally, the availability and size of STSGs are not limited by donor sites as they can be taken from almost any- where on the body including legs and trunk. FTSGs are harvested by direct excision and are limited in size as the donor site must be closed primarily. Donor sites for an FTSG are typically areas in which the skin can be spared; frequent sites include posterior auricular crease, groin, supraclavicular area, upper eyelids, and elbow crease.

C. Advantage. Because FTSGs retain all dermal appendages, they grow hair and secrete sebum to lubricate the skin and have the texture and appearance of normal skin. This feature makes FTSGs preferable for sites of cosmetic importance such as the face.

D. Advantage. The greater thickness of the FTSGs results in a more durable graft that is less subject to failure from external trauma.

E. Advantage. FTSGs undergo less scarring and secondary contracture, making them preferable for use over joints and on the hands to preserve mobility.

How well did you know this?
1
Not at all
2
3
4
5
Perfectly
7
Q
  1. Which of the following is not a suitable wound bed for a skin graft?
A. Muscle without overlying fascia
B. Muscle with overlying fascia
C. Tendon devoid of paratenon
D. Bowel with a layer of granulation tissue
E. Bone with overlying periosteum
A

C. Tendon devoid of paratenon

COMMENTS: A., B., D., and E. Suitable. As described in the previous question, wound beds that support skin grafts must be well vascularized with a healthy blood supply. Muscle is well vascular- ized and serves as an excellent site for skin grafts, with or without overlying fascia. Bone, tendon, and nerve all can support a skin graft, provided the periosteum, paratenon, and perineurium, respec- tively, are all intact. Skin grafts can be used to cover the bowel as a last resort for loss of abdominal domain. When the wound bed is suboptimal, STSGs are preferred because FTSGs have a higher failure rate in compromised sites as they are more susceptible to hematoma, vascular insufficiency, and infection.
C. Not Suitable. The paratenon must be present if the tendon is used as a wound bed for a skin graft.

How well did you know this?
1
Not at all
2
3
4
5
Perfectly
8
Q

Which of the following free flaps is incorrectly paired to its dominant blood supply?

A. Latissimus dorsi myocutaneous flap—thoracodorsal artery

B. Transverse rectus abdominus myocutaneous flap (TRAM)—superior epigastric artery

C. Fibular osteocutaneous flap—peroneal artery

D. Dorsalis pedis fasciocutaneous flap—anterior tibial artery

E. Tensor fascia lata flap—lateral femoral circumflex artery

A

B. Transverse rectus abdominus myocutaneous flap (TRAM)—superior epigastric artery

COMMENTS: A., C., D., and E. Correct.

A surgical flap involves the movement of a segment of tissue from one area to another to aid in closure of a wound or to provide soft tissue coverage.

This segment of tissue contains a vascular supply that may either be kept intact with its original inflow and advanced or rotated as a pedicle or the vascular supply may be transected and anastomosis performed at the new site; this is a free flap.

An island pedicle involves further skeletonizing and mobilizing the blood supply and allows for the tissue to be transferred to a site further away.

Free flaps can be supplied by a dominant pedicle, smaller multiple pedicles, or a combination of the two.

This question refers to muscle, fascial, or osseous flaps that are harvested utilizing a dominant vascular pedicle.

B. Incorrect. All are correctly matched to their dominant blood supply except for the TRAM flap. When performed as a pedicle flap, the superior epigastric artery is utilized; however, when the free TRAM flap is used, the vascular supply is the deep inferior epigastric artery, which is the dominant blood supply.

How well did you know this?
1
Not at all
2
3
4
5
Perfectly
9
Q
  1. Which of the following is incorrect regarding flaps?

A. Random skin flaps rely on blood supply from dermal and subdermal vascular plexus supplied by perforating arteries.

B. Axial flaps are named as such because the blood supply is a dominant vessel that runs longitudinally along the axis of the flap.

C. In practice, there are relatively few axial skin flaps as the majority of these are better described as fasciocutaneous flaps.

D. Random skin flaps involve geometric rearrangement and advancement of tissue and include examples such as rotation flap, Z-plasty, V-Y advancement, and rhomboid flap.

E. Fasciocutaneous flaps are the preferred choice of flap for a contaminated or osteomyelitic wound.

A

E. Fasciocutaneous flaps are the preferred choice of flap for a contaminated or osteomyelitic wound.

COMMENTS:

A. and B. Correct. Blood supply to a flap can be classified into random or axial. Random skin flaps derive blood supply from dermal and subdermal vascular plexus. Axial flaps are supplied by a dominant vessel that runs longitudinally along the axis of the flap. Because of this, random flaps are prone to necrosis at the distal extent of the flap, whereas axial flaps provide a more reliable blood supply for a greater length.

C. Correct. Tissue transferred via an axial flap may be skin, muscle, fascia, bone, nerves, bowel, or omentum. Axial skin flaps are more accurately characterized as fasciocutaneous flaps because they are supplied by vascular pedicles that originate from the deep fascia, emerge between the muscles, and travel in the intermuscular septum. These vessels, termed septocutaneous perforators, give off branches to an overlying cutaneous territory, and these territories dictate the design of fasciocutaneous flaps.

D. Correct. Random skin flaps include numerous types of local rearrangement and advancement of skin and subcutaneous tissue. V-Y advancement flaps advance skin on each side of a V-shaped incision to close a wound with a Y-shaped closure. Rota- tion flaps are semicircular flaps of skin in which the advancement is along the arc of the semicircle. Transposition flaps transpose skin around a pivot point to cover an adjacent defect, typically with an intervening segment of tissue. Z-plasty transposes two triangular flaps oriented as a Z-shaped design along a scar to lengthen or change the direction of the final resultant scar.

E. Incorrect. Myocutaneous flaps have an enhanced ability to eradicate infection and thus are the flap of choice in a wound with contamination or osteomyelitis.
Ref: 2

How well did you know this?
1
Not at all
2
3
4
5
Perfectly
10
Q
  1. Which of the following would most likely require free flap reconstruction?

A. Open wound at the knee with exposed total knee prosthesis

B. Open wound with exposed sternum following coronary artery bypass

C. Tumor excision involving full-thickness resection of the chest wall with exposed lung

D. Fracture of the distal third of the tibia with an open wound and exposed bone

E. Open fracture of the mid-humerus

A

D. Fracture of the distal third of the tibia with an open wound and exposed bone

COMMENTS: A., B., C., and E. Incorrect. Wounds described in these choices require muscle coverage but can be treated with a local muscle flap unless the flap has been used previously and failed. In that case, a free flap could be performed as a salvage procedure. For example, the knee can be covered with a pedicled gastrocnemius flap; the sternum can be covered with turnover or advancement pectoralis major flaps, pedicled rectus flap, or omental flap; the chest wall can be treated with a number of pedicled flaps from the trunk depending on the location of the wound; and the mid-humerus can be treated with fasciocutaneous flaps from the ipsilateral extremity.

D. Correct. Open tibial fractures have a high incidence of infection and nonunion, and thus tissue coverage is a critical com- ponent of treatment. Local muscle flaps are not dependable for defects on the distal third of the lower leg because they are usually
involved in the zone of injury, and there may not be adequate uninjured tissue available for local flap coverage. For this reason, free tissue transfer of muscle, skin, or both is the treatment of choice for defects of the distal third of the lower extremity.

How well did you know this?
1
Not at all
2
3
4
5
Perfectly
11
Q
  1. A 65-year-old woman with insulin-dependent diabetes mellitus develops sternal wound infection and dehiscence after coronary artery bypass that utilized the left internal mammary artery. Which of the following flaps is not an appropriate choice for coverage?

A. Bilateral pectoralis flaps

B. Omental flap based on left gastroepiploic vessel

C. Pedicle bilateral rectus abdominis flaps

D. Latissimus dorsi flap

E. Omental flap based on right gastroepiploic vessel

A

C. Pedicle bilateral rectus abdominis flaps

COMMENTS: A., B., D., and E. Appropriate. Pectoralis flaps are often used for sternal coverage and involve dissection of bilateral pectoralis muscles and advancement of the muscle bellies to the midline. Omental flaps can also be utilized, and some studies advo- cate omental flaps to have lower morbidity and mortality rates than myocutaneous flaps. These can be based off either the left or the right gastroepiploic artery. The latissimus dorsi flap can also be used and is especially useful for covering large anterolateral chest wounds or in cases where pectoralis flaps have been performed and failed.
C. Inappropriate Choice. All the flaps listed above are poten- tial options for tissue coverage for a sternal wound. However, in this patient, the left internal mammary artery has been transected and used for coronary artery bypass. This disrupts blood flow to the superior epigastric artery, the vascular pedicle for a rectus abdominis flap therefore making bilateral rectus flaps a poor choice.
Ref: 1, 7

How well did you know this?
1
Not at all
2
3
4
5
Perfectly
12
Q
  1. What is the most common cause of free flap necrosis?

A. Arterial thrombosis

B. Venous thrombosis

C. Arterial vasospasm

D. Arterial insufficiency secondary to technical error at the anastomosis

E. Infection

A

B. Venous thrombosis

COMMENTS: A., C., D., and E. Incorrect. In experienced hands, free flap success rates approach 95%. A well-controlled postopera- tive environment including a warm patient room and avoidance of tobacco (nicotine), caffeine, and vasoconstrictive medications are essential for success. Care must be taken for close postoperative monitoring of the flap typically in an intensive care unit setting, and identification of vascular compromise should be followed by prompt intervention. Arterial insufficiency is a less common cause of flap ischemia and manifests as a pale flap with the loss of capil- lary refill and reduction in temperature.
Compromise of vascular flow may be secondary to technical errors at the anastomosis, traction or kinking of the vessels, or compression by hematoma or edema. Return to the operating room (OR) is usually warranted with early intervention, allowing for the highest likelihood of flap salvage. Routine postoperative administration of antiplatelet agents or anticoagulants has not demonstrated superiority in the literature and is not a standard practice.
B. Correct. Venous thrombosis is the most common cause of free flap failure. A dusky-appearing flap with swelling, congestion, and rapid capillary refill are early signs of venous congestion.
Ref: 1, 3, 8–10

How well did you know this?
1
Not at all
2
3
4
5
Perfectly
13
Q
  1. Which of the following is false regarding the sequence of events in wound healing?

A. The initial vascular response is vasoconstriction, followed by vasodilation.

B. The phases of wound healing are inflammatory, proliferative, and remodeling.

C. The dominant cells of the inflammatory phase are macrophages.

D. The proliferative phase is mediated by fibroblasts.

E. The content of collagen in the wound continues to increase throughout the remodeling phase.

A

E. The content of collagen in the wound continues to increase throughout the remodeling phase.

COMMENTS: A., B., C., and D. True. Please see Comments of Question 2.
E. False. There is no net increase in collagen during the remodeling phase, only an increase in collagen cross-linking and conversion from type III to type I.
Ref: 1

How well did you know this?
1
Not at all
2
3
4
5
Perfectly
14
Q
  1. Which cell type is incorrectly matched to its role in the
    phases of wound healing?

A. Platelets—hemostasis and activation of inflammatory cells

B. Neutrophils—decontamination of the wound bed and amplification of the cellular response

C. Macrophages—phagocytosis and release of inflammatory factors

D. Lymphocytes—blood progenitor cells for tissue repair

E. Fibroblasts—collagen synthesis

A

B. Neutrophils—decontamination of the wound bed and amplification of the cellular response

COMMENTS: A. Correct. The initial response to injury is targeted at hemostasis and clot formation. This is achieved by platelet plug formation and activation of the coagulation cascade. To minimize blood loss, the tissue undergoes 5 to 10 min of vasoconstriction while the initial clot is formed. After this, mast cells and endothelial cells, through mediators such as histamine, prostaglandin E2, prostacyclin, and vascular endothelial growth factor, initiate vasodilation to usher in the cells of the inflammatory response. Inflammatory cells are first recruited via platelet degradation and release of platelet-derived growth factor and transforming growth factor-beta (TGF-β).
B. Incorrect. Neutrophils are the first to respond, the role of which is decontamination of the wound. However, neutrophils are not responsible for amplification of the cellular response; this is accomplished by macrophages.
C. Correct. Unless there is significant contamination, the mac- rophages become the dominant cells of the inflammatory response by days 2 to 3. The release of subsequent inflammatory factors and recruitment of cells to amplify the cellular response are mediated by macrophages, not neutrophils. The macrophages recruit addi- tional macrophages besides endothelial cells and fibroblasts. This
increase in cellular activity brings the wound healing into the pro- liferative phase.
D. Correct. Lymphocytes have a more active role in chronic wounds; however, in an acute wound, the contribution of blood progenitor cells aids in tissue repair.
E. Correct. Beginning around day 5, the main events of the proliferative phase are collagen deposition by fibroblasts and angio- genesis by endothelial cells. The collagen in the wound bed is primarily type I, with a more minor component of type III.
The remodeling phase begins around 3 weeks and continues up to a year after the injury. This phase involves prolonged synthe- sis and degradation of collagen but is marked by a steady state of collagen content. During the remodeling phase, the amount of type III collagen decreases and type I collagen increases, with the final ratio of type I:type III being 4:1. The density of capillaries in the wound decreases, and the scar becomes pale.
Ref: 1

How well did you know this?
1
Not at all
2
3
4
5
Perfectly
15
Q
  1. Which of the following statements is false regarding pressure ulcers?

A. Of all the layers of a wound, the subcutaneous tissue is the most susceptible to pressure necrosis due to its poor blood supply.

B. Stage I ulcers are defined by intact but erythematous skin that does not resolve after 1 h of pressure relief.

C. Stage II ulcers involve a partial-thickness loss of dermis with skin breaks or blistering, and stage III ulcers involve a full-thickness loss of dermis with exposed subcutaneous tissue.

D. Stage IV ulcers expose underlying muscle, tendon, or bone.

E. Ulcers where examination of the wound bed is obscured by a layer of slough or eschar are defined as unstageable.

A

A. Of all the layers of a wound, the subcutaneous tissue is the most susceptible to pressure necrosis due to its poor blood supply.

COMMENTS: A. False. The skin is more resistant to ischemia than the underlying tissues, which means that often the necrosis is more extensive than what is immediately apparent on examination. Muscle, though well vascularized, is most sensitive to ischemia. Therefore muscle is the most susceptible to pressure necrosis com- pared with skin and subcutaneous fat.
B., C., D., and E. True. Pressure ulcers occur when an unre- lenting external force of compression rises above the pressure gen- erated by the capillaries, resulting in ischemia. Capillary perfusion pressure is approximately 32 mmHg, and a recumbent body results in >80 mmHg in dependent areas. The areas at risk for developing pressure ulcers are scalp, sacrum, calcaneus when supine, ischium, and greater trochanter when sitting.
The stages described above are based on the staging system defined by the National Pressure Ulcer Advisory Panel. Stage I ulcers are defined by intact but erythematous skin that does not resolve after 1 h of pressure relief. Stage II ulcers involve a partial-thickness loss of dermis with skin breaks or blistering, and stage III ulcers involve a full-thickness loss of dermis with exposed subcutaneous tissue. Stage IV ulcers expose underlying muscle, tendon, or bone.
CHAPTER 32 / Plastic & Reconstructive Surgery 523

524 SECTION VIII / Subspecialties for the General Surgeon
The treatment of pressure ulcers is a correction of causative factors as well as debridement and procedures for tissue coverage when necessary. The Braden Scale for Predicting Pressure Sore Risk is a commonly used nursing tool and accounts for extrinsic and intrinsic factors that are most commonly the cause of pressure ulcers. It includes sensory perception, moisture, activity, mobility, nutrition, and friction and shear. When feasible, optimization of these risk factors is the first step in healing pressure ulcers. Stage I and II ulcers typically heal with local wound care. Stage III ulcers have the potential to heal, but this phase is often short lived and progresses rapidly to stage IV, which often requires excision of bony prominences and definitive reconstructive flap closure.
Ref: 1

How well did you know this?
1
Not at all
2
3
4
5
Perfectly
16
Q
  1. Which of the following statements is false regarding the formation of scars?

A. Hypertrophic scars are excessive scar tissue that remains within the boundaries of the initial tissue injury, whereas keloids extend beyond the area of injury.

B. A more cosmetic scar is achieved with incisions perpendicular to the lines of Langer or relaxed skin tension lines.

C. Relaxed skin tension lines usually run at right angles to the long axis of the underlying muscles.

D. Incisions on flexor surfaces usually heal well, whereas incisions over extensor joints heal with significant scarring.

E. In the remodeling phase of wound healing, collagen fibers become more organized, but they never achieve the precisely parallel arrangement seen in uninjured tissue.

A

ANSWER: B
COMMENTS: A. True. Hypertrophic scar and keloid scars are distinguished by the presence or absence of extent outside the confines of the initial incision or tissue injury. Hypertrophic scars generally regress with time and are treated with silicone gels and compression. Keloid scars are more difficult to treat, and some advocate the use of steroid injections or radiation to lessen the excessive scar tissue. Despite optimal treatment, keloid scars tend to recur.
B. False. Lines of Langer, also known as relaxed skin tension lines or wrinkle lines, represent the line of minimal skin tension, and thus incisions parallel to these lines have less tension on their closure and result in the most cosmetic scar.
C. True. If you were to excise a circular area of skin, this wound heals in the shape of an ellipse; the long axis of this ellipse defines a line of Langer. These lines typically run perpendicular to the long axis of the underlying muscle. To better understand this, imagine the transverse lines that form on the forehead and deepen when the frontalis muscle contracts. These lines are perpendicular to the long axis of the muscle, and incisions along these lines are under minimal tension.
D. True. Incisions over extensor surfaces are subject to increased tension with the movement of the joint, resulting in sig- nificant scarring. On the other hand, incisions on flexor surfaces are in areas of relaxed tension and heal with less scarring.
E. True. In the remodeling phase of wound healing, collagen fibers reorganize in a parallel arrangement, but they never achieve the same parallel arrangement seen in uninjured tissue.
Ref: 1

How well did you know this?
1
Not at all
2
3
4
5
Perfectly
17
Q
  1. A 35-year-old male is evaluated 24 h after a provoked dog bite to his left forearm. There is no suspicion for rabies. The wound is stellate and 2 cm deep. There is devitalized tissue as well as dirt at the base. His last tetanus booster was 7 years ago. Which of the following is the appropriate medical treatment?

A. Tetanus toxoid and antibiotic coverage for Eikenella corrodens

B. Antibiotic coverage for Pasteurella multocida only

C. Tetanus toxoid and antibiotic coverage for P. multocida

D. Tetanus toxoid, tetanus immune globulin, and antibiotic coverage for P. multocida

E. Tetanus toxoid only

A

C. Tetanus toxoid and antibiotic coverage for P. multocida

COMMENTS: A., B., D., and E. Inappropriate. To determine the appropriate treatment, the wound must be divided into tetanus prone and non–tetanus prone. The characteristics of tetanus-prone wounds include the following: older than 6 h; stellate configuration or avulsion; depth greater than 1 cm; gross contamination or infec- tion; devitalized tissue or necrosis; or the result of crush injury, burn, or frostbite. If the patient has previously received an initial tetanus and diptheria toxoids vaccine and at least three doses of tetanus toxoid or booster, an additional tetanus booster is required only if his last dose was more than 5 years ago. If the patient has not completed the initial vaccine and three boosters, or his immu- nization status is unknown, he should receive tetanus immune globulin and tetanus toxoid.
For non–tetanus-prone wounds, tetanus immune globulin is not indicated. Tetanus toxoid is indicated only if the patient did not previously complete the vaccine and three boosters, completed the immunization course but the last booster was more than 10 years ago, or the immunization status is unknown.
C. Appropriate. This patient requires a tetanus booster because his last dose was more than 5 years prior to injury. Antibiotics prophylaxis is usually administered for animal and human bites, especially if there is delayed presentation for more than 12 h; the patient is diabetic or otherwise immunosuppressed; or there is involvement of face, hands, or feet. Although bite wounds are often polymicrobial, P. multocida is the major pathogen in dog and cat bites. Human bites are often contaminated with E. corrodens, Staphylococcus aureus, S. epidermidis, α- and β-hemolytic Strep- tococcus sp., and Corynebacterium. Amoxicillin–clavulanate is the drug of choice for most bite wounds because it has aerobic and anaerobic coverage, including that for Pasturella sp. and Eikenella sp.
Ref: 11, 12

How well did you know this?
1
Not at all
2
3
4
5
Perfectly
18
Q
  1. A 60-year-old diabetic woman currently being treated for right lower extremity cellulitis is brought to the emergency room from her nursing home. She has an exquisitely tender erythematous right leg with bullae formation. She has tenderness that extends beyond the area of erythema. Her body temperature is 38.9°C, blood pressure is 82/43 mmHg, and heart rate is 128 beats/min. After initiating fluid resuscitation, what is the next best step in management?

A. Admission to the intensive care unit (ICU), blood cultures, broad-spectrum antibiotics, and serial examinations

B. Computed tomography (CT) scan of the right lower extremity

C. Admission to the ICU, blood cultures, broad-spectrum antibiotics, and bedside debridement of the bullae of the right lower extremity

D. Blood cultures, broad-spectrum antibiotics, and immediate transfer to the OR for debridement of the right lower extremity

E. Blood cultures, broad-spectrum antibiotics, and IV hydrocortisone

A

D. Blood cultures, broad-spectrum antibiotics, and immediate transfer to the OR for debridement of the right lower extremity

COMMENTS: A., B., C., and E. Incorrect. The clinical scenario is suggestive of a necrotizing soft tissue infection of the extremity, which is often characterized by erythema and severe pain out of proportion to physical examination findings. Additional signs include tenderness beyond the area of erythema, crepitus, bullae, and skin necrosis. Patients often demonstrate signs of sepsis with fevers, tachycardia, hypotension, changes in mental status, and oliguria/anuria. In stable patients or in cases where the diagnosis is not clear, imaging with CT, magnetic resonance imaging (MRI), or plain films may be useful and may demonstrate subcutaneous edema or emphysema and inflammation and fat stranding. Soft tissue gas confirms a necrotizing infection, but its absence does not exclude the diagnosis.
D. Correct. The appropriate treatment is the initiation of resus- citation, blood cultures, broad-spectrum antibiotics, and immediate operative debridement. In the OR, all necrotic and infected tissue should be aggressively debrided, and consent should be obtained from patients or families for possible amputation when applicable. Wide drainage with Penrose drains may also be appropriate. Often, multiple debridements are required, leading to large debilitating wounds requiring extensive reconstruction.
Ref: 13–15

How well did you know this?
1
Not at all
2
3
4
5
Perfectly
19
Q
  1. In addition to broad-spectrum antibiotics, such as vancomy- cin, and a carbapenem, piperacillin/tazobactam, or an aminoglycoside, what other antibiotic should be given to the above patient?

A. Metronidazole

B. Clindamycin

C. Azithromycin

D. Cefazolin

E. Cefepime

A

B. Clindamycin

COMMENTS: A., C., D., and E. Incorrect. For necrotizing soft tissue infections, antibiotic therapy should include coverage for gram-positive, gram-negative, and anaerobic bacteria. There are
many acceptable regimens such as a carbapenem, piperacillin/tazo- bactam, or tigecycline. An appropriate multidrug regimen is high- dose penicillin and a fluoroquinolone or aminoglycoside. Vancomycin should be included until methicillin-resistant S. aureus (MRSA) is excluded as a causative organism. Metronidazole, azithromycinn, cefazolin, and cefepime would not provide com- plete coverage.

B. Correct. Clindamycin should also be administered. Clindamycin is a protein synthesis inhibitor and thus is useful for inhibition of toxic production, especially in Clostridium sp. and group A β-hemolytic Streptococcus sp. Once adequate source control is achieved with debridement and the patient clinically stabilizes, antibiotic administration can be discontinued.
CHAPTER 32 / Plastic & Reconstructive Surgery 525
8.
Ref: 16

20
Q

Which of the following is an acceptable wound environment for a negative-pressure wound dressing or vacuum-assisted wound closure?

A. Dehiscence after inguinal lymph node dissection with exposed femoral vessels

B. Sternal dehiscence and mediastinitis following cardiac surgery

C. Stage IV sacral decubitus ulcer with untreated sacral osteomyelitis

D. Lower extremity wound with necrotic muscle in base

E. Lower extremity wound with exposed prosthetic graft

A

B. Sternal dehiscence and mediastinitis following cardiac surgery

COMMENTS: A. and E. Incorrect. Negative-pressure wound therapy (NPWT) is contraindicated in wounds with exposed vas- culature, vascular grafts, or prosthetic grafts that may cause erosion leading to hemorrhage or infection. However, NPWT can be used in wounds with vessels or grafts that have adequate tissue coverage.
B. Correct. Vacuum-assisted wound closure involves the application of a porous sponge and occlusive dressing to the wound followed by a device to generate negative pressure over the wound surface. NPWT accelerates wound healing and granulation tissue formation by increasing blood flow, removing exudate, and provid- ing mechanical forces to draw the wound edges together. NPWT may be used successfully for chest coverage in cases of sternal dehiscence after adequate debridement as the primary treatment or a bridge to definitive therapy with flap coverage.
C. and D. Incorrect. NPWT should only be utilized on wounds that have been properly debrided and are free of necrotic or infected tissue. Therefore NPWT is inappropriate in osteomyelitis if antibi- otic therapy and possible debridement have not been completed.

21
Q
  1. When counseling a young woman about breast surgery, she states that she has been doing some Internet research, and she voices many concerns about implants. Which of the following statements is true?

A. Women with breast implants are at an increased risk of developing breast cancer.

B. When a woman with breast implants is diagnosed with breast cancer, it is more likely to be at a more advanced stage compared with women without breast implants.

C. Thoroughly evaluating the breast tissue in a breast with an implant is more difficult using standard mammography.

D. The concentration of silicone in breast milk is higher in women with breast implants than in those without implants.

E. Studies have demonstrated a causal relationship between breast implants and connective tissue disorders such as lupus and rheumatoid arthritis.

A

C. Thoroughly evaluating the breast tissue in a breast with an implant is more difficult using standard mammography.

COMMENTS: A. False. No evidence exists to suggest that breast implants impart a higher risk of breast cancer.
B. False. There is no significant difference in size or stage of tumor at the time of detection between women with and without implants.
C. True. In women with implants, standard mammography is able to evaluate only 75% of the breast tissue, with the remaining tissue obscured by the implant. This requires specialized views utilizing displacement techniques called Eklund views to obtain additional images. Studies demonstrate that this does not result in delayed detection or poorer prognosis in women with implants.
D. False. Depending on the location of the implant, breast- feeding with implants may still be possible. Investigations indicate no significant difference in the concentration of silicone in breast milk in women with implants.
E. False. Case reports published in the 1980s generated concern about a relationship between breast implants and the devel- opment of connective tissue disorders. Over 2000 peer-reviewed studies were analyzed and demonstrated no definitive evidence of a link between implants and connective tissue disorders or that women with implants were more likely than the general population to develop these diseases.
Ref: 1

22
Q
  1. Regarding postmastectomy reconstruction, which of the following is false regarding the advantages and disadvantages of implant reconstruction?
    A. Although single-stage implant reconstruction allows for a
    single operation, implant size is often limited by the skin
    envelope.
    B. Tissue expanders allow for a larger size of the final implant but require frequent office visits for expansion and a second operation.
    C. Two-stage implant reconstruction often requires 4 to 6 months for completion of the process.
    D. Excessive scar tissue may form around an implant. This is termed capsular contracture and may result in defor- mity and pain.
    E. Because it is placed under sterile conditions and bacteria is not typically present in breast tissue, there is virtually no risk of implant infection.
A

ANSWER: E
COMMENTS: A. True. Single-stage implant reconstruction involves the placement of the final implant at the time of mastec- tomy. With this technique, the size of the skin envelope at the conclusion of the mastectomy dictates the maximum size of the implant. Placing too large an implant with an inadequate skin envelope leads to excess tension on the closure and increases the risk for wound dehiscence and skin necrosis.
B. True. Two-stage implant reconstruction is performed in two operations. The first stage involves placement of a temporary tissue expander, often at the time of mastectomy, and the second stage involves removal of the expander and placement of the final implant. Tissue expansion allows a larger size of the final implant than that allowed by the single-stage implant reconstruction.
C. True. The tissue expander consists of a silicone envelope with an injection port that can be accessed percutaneously and is expanded with saline over time. This typically requires weekly office visits until the skin achieves the desired size of the final implant. Surgeons often delay the second stage of the operation until 1 to 4 months after the completion of expansion.
D. True. Capsular contracture occurs in approximately 15% of patients and is usually apparent by 6 months after placement of the implant, although it may occur at any time over the lifetime of the implant. The incidence is lower with the use of a textured implant than with the use of a smooth shell implant. Implant massage exercises in the early postoperative period may reduce the risk of contracture development. If the fibrous tissue is significant, it may result in a painful or esthetically deformed breast often requiring surgical correction.
E. False. Like all prosthetic materials, implants are subject to infection, although this is an uncommon occurrence. Bacteria present in the breast ducts are similar to normal skin flora. A meticulous aseptic technique must be utilized to reduce the risk of implant infection. In some cases, IV antibiotics can salvage the implant, but infection of the implant may ultimately necessitate removal of the implant. The implant may be replaced after adequate resolution of the infection.
Ref: 1, 2

23
Q
  1. You are counseling a young breast cancer patient regarding her options for reconstruction after mastectomy. She states that she does not want any artificial implants and would prefer the use of only her own tissues for reconstruction. Which of the following statements is false?

A. If the patient has adequate abdominal tissue, a TRAM flap is an excellent option and has the added benefit of a modified abdominoplasty.
B. A TRAM flap offers the benefit of increasing or decreas- ing the volume accordingly if the patient gains or loses weight.
C. A disadvantage of the pedicle TRAM flap is the potential risk for abdominal hernias and abdominal wall laxity.
D. A latissimus dorsi myocutaneous flap is a reasonable option for this patient and has the benefit of avoiding abdominal donor site morbidity.
E. Deep inferior epigastric perforator flap is a reasonable option for this patient and has the benefit of avoiding abdominal donor site morbidity.

A

ANSWER: D
COMMENTS: A. True. TRAM flap reconstruction is the most commonly performed autologous tissue breast reconstruction. The rectus muscle and supplying superior epigastric artery serve as a vascular pedicle, and a flap of lower abdominal skin and fat is transferred to the mastectomy defect to reconstruct the breast. The muscle remains attached at the proximal origin and is tunneled superiorly to reach the chest. The abdominal defect is closed by reapproximation of the anterior rectus sheath and advancing the superior skin edge of the abdominal skin inferiorly for closure. This results in a modified abdominoplasty that many surgeons and patients regard as an added benefit. This flap generally provides more than adequate volume for reconstruction of the breast and can be utilized for unilateral or bilateral breast reconstruction.
In contrast to tissue expander and implant reconstruction, the TRAM flap is a single-stage operation that can be performed at the time of mastectomy. However, this is a considerably longer opera- tion than either stage in the two-stage technique and requires a longer recovery time.
B. True. Both the TRAM and deep inferior epigastric perfora- tor (DIEP) flaps are composed of native abdominal tissue and maintain proportionality to the overall body mass and contralateral breast and expand or decrease in volume when a patient gains or loses a significant amount of weight.
C. True. The resulting muscle defect does carry a potential risk for abdominal hernia and abdominal wall laxity, resulting in an undesirable bulge of the lower abdomen. The incidence of this is up to 5%. Often, the abdominal fascia is reinforced with the acel- lular dermal matrix or prosthetic mesh. This complication may be avoided with DIEP free flap reconstruction.
D. False. A latissimus dorsi myocutaneous flap is a pedicle flap based on the thoracodorsal vessels and is tunneled anteriorly for chest
wall coverage. It is particularly useful for providing coverage in patients who have a pectoralis muscle defect after radical mastectomy or when mastectomy skin flaps are inadequate. This flap often has insufficient volume to fully reconstruct the breast and is used in com- bination with either single- or two-stage implant reconstruction.
E. True. In the DIEP free flap reconstruction, a flap of lower abdominal skin and fat is harvested based on the deep inferior epigastric artery with little to no rectus abdominis muscle. This flap is then transferred to the chest with free tissue transfer. The DIEP flap is anastomosed to a recipient vessel, usually thoracodorsal, subscapular, or internal mammary. This technique allows for ade- quate breast reconstruction with autologous tissue but minimizes the abdominal wall morbidity at the donor site.
CHAPTER 32 / Plastic & Reconstructive Surgery 527

24
Q
4.
Ref: 1, 2
A 35-year-old woman who underwent augmentation mammoplasty with silicone breast implants notices a flattening of her left breast and is worried about the rupture of her implant. Which modality is the most sensitive and specific for detection of implant rupture?
A. Physical examination B. Ultrasound
C. CT scan
D. Mammography
      E. MRI
A

ANSWER: E
COMMENTS: A. Incorrect. Many implant ruptures are clinically silent and detected only at the time of implant exchange or with routine imaging.
E. Correct. MRI has a sensitivity and specificity of greater than 90% for detecting implant rupture. Intracapsular rupture is identified on MRI by the so-called linguine sign in which multiple low-density lines represent the implant shell folding on itself. Extracapsular rupture is diagnosed by high-intensity focus outside the capsule, representing extravasation of the silicone gel into the tissues surrounding the capsule. Because the cohesive gel often remains in the vicinity of the ruptured implant, it may be clinically silent. It is thus recommended to undergo regular imaging surveil- lance of the implant, and current recommendations advise MRI 3 years after the implant placement and then every 3 to 5 years for the life of the implant. If rupture occurs, removal of the implant with or without replacement is recommended. Rupture of a saline implant is clinically easier to detect with deflation of the implant.
B., C., and D. Incorrect. Mammography, ultrasound, and CT scan have all been used but are less reliable for detection.
Ref: 20–22

25
Q
  1. When performing a component separation for a large abdominal wall defect, which of the following describes the location of the relaxing incision?
    A. The aponeurosis of the internal abdominal oblique, at the linea semilunaris
    B. The aponeurosis of the external abdominal oblique, 1 to 2 cm lateral to the linea semilunaris
    C. The anterior rectus sheath, 1 to 2 cm medial to the linea semilunaris
    D. The aponeurosis of the internal abdominal oblique, 1 to 2 cm lateral to the linea semilunaris
    E. The aponeurosis of the external abdominal oblique, 1 to 2 cm medial to the linea semilunaris
A

ANSWER: B
COMMENTS: B. Correct. Separation of components is a tech- nique that utilizes a series of abdominal fascial releases to achieve adequate mobilization of the tissues to close a large midline mus- culofascial defect of the anterior abdominal wall. Via a vertical midline incision, the skin and subcutaneous fat are dissected off the muscular wall.
The aponeurosis of the external abdominal oblique is tran- sected longitudinally, 1 to 2 cm lateral to the semilunaris, and this incision is extended from the pubis onto the chest wall, at least 5 to 7 cm cranial to the costal margin. A plane of dissection is carried out laterally between the external and internal abdominal obliques as far lateral as possible, ideally to the posterior axillary line. In order to avoid wound complications and necrosis of skin flaps, care should be taken to preserve large perforating vessels that supply the skin and subcutaneous tissue.
The neurovascular supply to the muscle is not endangered by this technique as it runs between the internal oblique and the trans- versus abdominis. The relaxation and dissection allow for medial mobilization of the rectus and the internal oblique and primary closure of the two rectus muscles in the midline. When needed, an additional 2 to 4 cm of mobilization can be gained by separating the rectus from the posterior rectus sheath through an incision of the posterior sheath while taking care to preserve the blood supply that enters posterolaterally between the internal oblique and the transversus abdominis. Separation of components on both sides of the abdominal wall in conjunction with relaxing incisions in the posterior rectus sheath can yield up to 20 cm of medial advancement.
A., C., D., and E. Incorrect. Further relaxing incisions to the internal oblique or transversus abdominis have been described, but this can result in lateral bulge or herniations. Many surgeons advo- cate the use of biologic or prosthetic mesh as either an underlay or an onlay to reinforce the repair and reduce the rate of recurrence. Minimally invasive separation and laparoscopic/endoscopic sepa- ration of components have also been described.
Ref: 1, 2

26
Q
  1. A 45-year-old male is recovering on postoperative day 1 in the surgical ICU following a large separation of components
    operation. Due to the prolonged length of the operation, he received a high volume of intravenous fluids in the OR and was left intubated at the conclusion of the case. Which of the following is inconsistent with the diagnosis of abdominal compartment syndrome?
    A. Oliguria or anuria
    B. Bladder pressures exceeding 20 to 25 mmHg
    C. Peak airway pressures > 45 cmH2O
    D. Intraabdominal pressure greater than 16 mmHg
    E. Elevated central venous and pulmonary artery pressures
A

ANSWER: D
COMMENTS: A. Consistent. Resultant renal dysfunction occurs from both decreased renal perfusion and venous congestion from compression of the renal vein. This combination of increased parenchymal pressure and decreased renal blood flow decreases the pressure gradient across the glomerular membrane and therefore reduces the glomerular filtration rate (GFR). The same effects of decreased perfusion and increased venous congestion result in mucosal ischemia of other visceral organs and lactic acidosis.
B. Consistent. The most widely used method for monitoring abdominal pressures is the measurement of bladder pressures via a transducer on a Foley catheter. Under normal conditions, bladder pressure and thus the intraabdominal pressure in a supine adult are less than 10 mmHg. Following abdominal surgery, pressures range from 3 to 15 mmHg. Pressures exceeding 20 to 25 mmHg, along with other signs, should raise concern for abdominal compartment syndrome.
Recognition of abdominal compartment syndrome is critical, and management includes judicious administration of IV fluids, adequate sedation, and neuromuscular paralytics. When conserva- tive management fails, a decompressive laparotomy is indicated.
C., E. Consistent. As the pressure in the abdominal cavity increases, the diaphragm is elevated and results in the transmission of the high pressure into the thoracic cavity and significant reduc- tion in pulmonary compliance. This is manifested by an increase in the peak inspiratory pressures required for adequate ventilation with peak airway pressures > 45 cmH2O. The increased intratho- racic pressure results in decreased venous return, which, com- pounded by increased systemic vascular resistance from mechanical compression of capillary beds, reduces cardiac output. The high thoracic pressures are reflected in elevated central venous and pul- monary artery pressures.
D. Inconsistent. Following large abdominal wall reconstruc- tion, the decreased volume of the abdominal cavity (decreased abdominal domain) can lead to the development of increased intraab- dominal pressures. This is exacerbated by high-volume fluid resus- citation and edema, resulting in decreased compliance. Normal intraabdominal pressure is 5 to 7 mmHg in critically ill adults. Abdominal hypertension is defined as sustained pressures greater than 12 mmHg, and abdominal compartment syndrome is a sustained pressure greater than 20 mmHg with associated organ dysfunction.
Ref: 1, 23

27
Q
  1. You are called to evaluate a 25-year-old female who has sustained a lip laceration from a dog bite. Both the lip mucosa and the adjacent skin are involved. Which of the following statements is false regarding the repair?
    A. The most critical steps are exact alignment of the vermilion border and repair of the orbicularis oris muscle.
    B. If the defect results in loss of up to one-third of the lip, it can be repaired primarily.
    C. If the defect is large and to the upper lip, it can be repaired with a tissue flap from the lower lip.
    D. If the defect is large and to the lower lip, it can be repaired with a tissue flap from the upper lip.
    E. If the repair results in microstomia, this is often temporary.
A

ANSWER: D
COMMENTS: A. True. Repair of a lip laceration that disrupts the vermilion, or mucocutaneous junction of the lip, requires precise alignment as even small irregularities in the border will be notice- able. Repair of injury to the orbicularis oris muscle is necessary to maintain the competence of the lip.
B. True. Primary repair is usually possible for defects up to one-third of the lip, and if full thickness, it should be performed in three layers from deep to superficial: mucosa, muscle, and skin.
C. True. For large defects of the upper lip, a tissue flap from the lower lip can be used. An example of this is the Abbe flap, which is a pedicle flap of the lower lip based on the labial artery. The flap is rotated to the upper lip, and the pedicle is divided 2 to 3 weeks later, after the flap has established a new blood supply. The lower lip is well suited to this purpose because it does not have any anatomic landmarks nor central structure. Therefore its appear- ance is not affected by the loss of tissue or asymmetry.
D. False. The upper lip, however, has the so-called Cupid’s bow and central tubercle, and loss or displacement of these midline structures results in significant asymmetry. Because of this, the upper lip cannot be used as a flap donor site for repair of a defect of the lower lip. In the case of large defects of the lower lip, cheek advancement flaps can be utilized to gain enough mobility to close the defect. For the full loss of the lower lip, free flap reconstruction using the free radial forearm flaps with palmaris longus tendon can be used.
E. True. The flaps are full thickness up to the commissures, where the flaps become superficial to the muscles to preserve the labial vessels and nerve. This preserves oral competence but often results in microstomia, which is typically temporary as the tissues stretch over time.
Ref: 1,26

28
Q
  1. A 12-month-old boy with cleft lip and palate presents to the clinic for evaluation. The child underwent repair of a cleft lip at the age of 3 months. On examination, the palatal cleft involves both the hard and soft palate and is continuous with an alveolar defect. There is also asymmetry of the widened
    nose and deviation of the nasal septum. What is the best operative plan for this child’s next surgery?
    A. Closure of the palatal defect
    B. Alveolar bone graft
    C. Closure of the palatal defect and alveolar bone graft D. Correction of the septal deviation
    E. Correction of the septal deviation and rhinoplasty
A

ANSWER: A
COMMENTS: A. Correct. The overall incidence of cleft lip and palate is 1 in 750 children, with a higher incidence, 1 in 300, in the Asian population. The deformity can be isolated cleft lip (21%), isolated cleft palate (33%), and combined cleft lip and palate (46%). The defect has a complex genetic inheritance pattern, and the pathophysiology remains incompletely understood. The repair of cleft lip and palate is performed in a multistaged fashion and typically requires three to four operations, with additional opera- tions to correct persistent anomalies.
The deformity can be bilateral or unilateral and is considered complete if it extends into the nose and results in an absence of the nasal floor. The cleft can extend into the gum and through the alveolus, resulting in a bony defect. The cleft can involve only the soft palate, a result of the abnormal insertion of the levator palatini muscle on the posterior surface of the hard palate, or involve the hard palate as well. In some cases, the mucosa over the palate may be intact, and there is a separation of the underlying musculature (submucous cleft).
The first operation is repair of the cleft lip. There is no exact consensus on when this should be performed, but this operation is typically performed between 3 and 6 months of age. This allows for safer administration of anesthesia due to the weight gain in these first 3 months. The next step in reconstruction is the closure of the palatal defect. This may involve simple reapproximation of the muscle and closure of the palatal mucosa. However, wide unilateral clefts and bilateral clefts often require relaxing incisions and release of the mucosa from the underlying bone. Delay in this phase allows for the better growth of the midface and palate as dissection of the mucosal tissue off the bone restricts growth. However, early repair of the palate improves speech development. Therefore a balance must be found between the two. Palate repair is most often per- formed around 1 year to 18 months of age.
B. and C. Incorrect. Bone grafting is often performed at 7 to 10 years of age at mixed dentition stage. Correction of an alveolar cleft must be undertaken with a multidisciplinary approach that includes an orthodontist. Bone grafting to the alve- olar cleft restores the dental arch, which is necessary for tooth emergence. The orthodontist monitors for tooth eruption and may initiate early therapies to align the teeth in preparation for bone grafting. The operation involves excision of the oral–nasal fistula, reconstruction of the adjacent nasal floor, placement of a bone graft in the alveolar defect, and placement of gingival flaps over the graft. The bone graft consists of cancellous bone and can be harvested from the iliac crest or the cranial diploe, which is the layer of cancellous bone between the inner and outer layers of the cranium.
CHAPTER 32 / Plastic & Reconstructive Surgery 529

530 SECTION VIII / Subspecialties for the General Surgeon
D. and E. Incorrect. The last operation is septorhinoplasty, and this is usually performed in the teenage years, after maturation of dentition and completion of the orthodontic work. A deviated septum is almost always a component of cleft lip and palate and is straightened in this operation. Rhinoplasty is tailored to each par- ticular patient but often involves repositioning and trimming of nasal cartilage, in-fracture of the nasal bones to correct a widened nose, and tacking of the lower nasal cartilage to improve the projec- tion of the nose.
Ref: 1, 2

29
Q
  1. The mother of a 4-year-old boy with a history of cleft lip and palate repair brings her child to the clinic due to concerns regarding his speech. The child underwent lip closure at 3 months of age and palate closure at 12 months. He has been participating in speech therapy since the palatal closure but continues to have speech difficulties and produces squeaks and snorts through his nose with attempted speech. On examination, you note an alveolar cleft. The mother states she is agreeable to another operation if it exists. What is the best plan to correct the problem for this patient?
    A. Continued speech therapy as there are no operative methods for correction
    B. An operation to close the persistent oral–nasal fistula
    C. Placement of a bone graft at the alveolar defect
    D. An operation to place tissue near the velopharyngeal port
    E. Operative correction of the septal deviation and rhinoplasty
A

ANSWER: D
COMMENTS: A. Incorrect. Primary repair of the palatal cleft normalizes speech in only approximately 80% of children, and the remaining 15%–20% continue to demonstrate velopharyngeal insufficiency. This is the result of an inability of the velum, or soft palate, to seal off the connection to the nasal cavity during speech. This causes hypernasality of the speech and escape of air through the nose during attempted pronunciation of certain consonants. Speech therapy may be inadequate to correct this, and operative intervention may be indicated.
B. Incorrect. The persistent oral–nasal fistula is at the site of the alveolar defect and is not the cause of the speech abnormalities described.
C. and E. Incorrect. Both alveolar bone grafting with the closure of the oral–nasal fistula and correction of the deviated septum are performed later in life and would not result in correction of the problems with speech.
D. Correct. The surgery aims to correct the velopharyngeal insufficiency. A number of techniques exist (superior pharyngeal flap, sphincter pharyngoplasty, augmentation pharyngoplasty), but the objective is to improve closure of the velopharyngeal port to aid in the regulation of airflow and the ability to impede the move- ment of air into the nasopharynx.
The timing of this operation must be early enough to have maximum impact on speech development but late enough to give adequate time for speech therapy to attempt correction of the problem. The consensus is operative intervention after 3 years of age. Prior to an operation, the dynamics of the child’s speech should be thoroughly evaluated via nasoendoscopy and functional mea- sures obtained with nasometry to determine the best treatment approach.
Ref: 1, 2

30
Q
  1. Which of the following statements is true regarding vascular anomalies?
    A. Newborns with capillary malformations (port wine stains) in the ophthalmic distribution of the trigeminal nerve should undergo MRI of the head.
    B. A 3-year-old with a stable hemangioma on the forehead that has failed to regress should be treated with surgical excision.
    C. Hemangiomas are typically present at birth.
    D. Most venous malformations regress and therefore should be observed until age 5 to 7 years.
    E. Sclerotherapy is usually effective for the treatment of arteriovenous malformations.
A

ANSWER: A
COMMENTS: A. True. When a capillary malformation occupies the ophthalmic trigeminal nerve distribution, MRI of the head should be obtained to evaluate for Sturge-Weber syndrome, which is associated with ipsilateral leptomeningeal and ocular anomalies.
Vascular anomalies are divided into two groups: tumors and malformations. Vascular tumors arise as a result of increased pro- liferation of endothelium and include hemangiomas (most common), hemangioendotheliomas, tufted angiomas, and heman- giopericytomas. Malignant vascular tumors, such as angiosarco- mas, are rare in infancy.
In contrast, vascular malformations result from abnormal arte- rial, capillary, venous, or lymphatic components. These malforma- tions may consist of a single component or a combination of components and may be high flow, low flow, or mixed.
B. and D. False. There is typically complete regression of hemangiomas in 50% of cases by the age of 5 years and in 70% of cases by the age of 7 years. Observation is appropriate unless the tumor affects the vision or involves the airway; in such cases inter- vention is indicated. A course of injected or systemic steroids may induce involution, but if this fails, operative resection should be performed. Beta-blockers have shown to be effective in decreasing the proliferative phase of hemangiomas and may be utilized topi- cally or systemically.
Lymphatic malformations, also known as lymphangiomas or cystic hygromas, are the most difficult to treat because they are diffuse and often adjacent to vital structures in the head and neck. Lymphangiomas expand and contract with alterations in the flow of lymphatic fluid. If functionally impairing, these are to be removed via a series of staged partial excisions.
C. False. Hemangiomas are not present at birth and typically develop in the first 2 weeks of life; these are commonly referred to as strawberry angiomas. They are typically characterized by a rapid growth phase followed by slow spontaneous involution.
On the other hand, vascular malformations are usually present at birth and often grow in proportion with the child. Initially a flat lesion, these malformations can become thick and verrucous as they enlarge and there is no spontaneous regression. Early treatment is indicated and depends on the vascular components involved. Small capillary malformations, such as port wine stains, are well treated with pulse dye laser, and large lesions may require combined laser and operative therapy.
E. False. Sclerotherapy is the standard treatment for venous malformations and may be a useful adjunct in arteriovenous mal- formations. However, due to the development of collaterals, sclero- therapy alone is often insufficient for arteriovenous malformations

and surgical resection should be performed after sclerotherapy and embolization.
Ref: 1, 2

31
Q
5. A concerned father brings his 7-month-old female to the clinic for evaluation. He is concerned that her skull is misshapen and not growing properly. On examination, the brow above the baby’s right eye is recessed, and the left brow is overly protuberant. You suspect she has craniosynos- tosis. Which suture is most likely affected in this patient?
A. Sagittal suture
B. Metopic suture
C. Right coronal suture
D. Left coronal suture
E. Bilateral coronal sutures
A

ANSWER: C
COMMENTS: A. Incorrect. Fusion of the sagittal suture, which is the most commonly affected by craniosynostosis, develops a long oval-shaped head, which is often described as boat or keel shaped or scaphocephaly.
B. Incorrect. Synostosis of the metopic suture results in a triangular forehead and is known as trigonocephaly.
C. Correct. Craniosynostosis is characterized by premature fusion of one or more cranial sutures and resultant restriction in the growth of the skull. Although the fontanelles are obliterated early in life, the sutures remain open to allow for expansion of the brain and skull. With the exception of the metopic suture that closes around the age of 3 to 9 months, the remaining sutures do not fuse until adulthood.
When early synostosis occurs, the growth of the skull is restricted in the direction perpendicular to the affected suture and compensatory growth occurs in a parallel direction to the suture. Multiple sutures can be affected and produce a combination of the shapes described. This syndrome may result in solely a physical deformity or may cause enough restriction to cause increased intra- cranial pressure, and brain development may be affected. Unilateral coronal suture involvement produces a recessed supraorbital bar on the affected side and compensatory growth of the contralateral brow; this is referred to as frontal plagiocephaly.
Surgical treatment aims to correct the deformity, restore pro- jection to the brow for the protection of the eyes, and relieve or prevent increased intracranial pressure. This is most often per- formed within the first year of life. The operation is typically per- formed through a sinusoidal coronal incision, and although many operative techniques exist, the objective is release or excision of the fused suture. In the first 2 years of life, the dura is osteogenic and gaps in the skull from the operation slowly ossify over time.
D. Incorrect. Premature fusion of the left coronal suture results in compensatory growth of the right brow, resulting in a recessed left eye and protuberant right brow.
E. Incorrect. Bilateral coronal synostosis results in a shortened and flattened forehead, termed brachycephaly.
Deformational plagiocephaly should be included in the dif- ferential diagnosis and is caused by supine sleeping. This does not require operative intervention and will often “self-correct” as the child grows and is able to turn during sleep. Helmet molding therapy can be considered in these children to improve cosmesis.
Ref: 2

32
Q
  1. A 15-year-old female presents to the clinic for evaluation. She has a history of bilateral cleft lip and palate and has
    undergone all stages of repair, including alveolar bone grafting. She has completed orthodontic therapy and has all of her permanent teeth. She is concerned because the middle of her face is recessed and her upper and lower teeth do not align well, with the lower teeth projecting further than the upper ones. What is the best management of this condition?
    A. Refer her back to the orthodontist for realignment of the teeth.
    B. Refer her to physical and occupational therapy for techniques to maximize function.
    C. Perform fat grafting to the midface to fill in the recessed areas and reshape the face.
    D. Perform an operation to advance the maxilla forward.
    E. Correct the recessed midface with a local advancement
    flap or a free flap.
A

ANSWER: D
COMMENTS: A. Incorrect. The growth of the midface may be compromised by palatal closure, especially if there is an extensive dissection of the mucosa from the bone as is necessary for large clefts. Midface hypoplasia is characterized by a recessed maxilla, which is especially apparent in lateral views of the patient’s face. This can result in an overly projectile appearance of the eyes and malocclusion of the mandible.
If there is an esthetic or functional impairment to the patient, operative intervention can restore normal occlusion. This operation should be delayed until the orthodontic work is complete, skeletal maturity has been reached, and the dentition is in its final position. This is most commonly performed in the teenage years. This patient has already completed orthodontic work; referral back to the ortho- dontist will not correct this condition.
B., C., and E. Incorrect. Physical therapy, occupational therapy, fat grafting, and flap reconstruction will not correct the patient’s midface hypoplasia.
D. Correct. The most common operative correction is a Le Fort I osteotomy, where the maxilla is incised horizontally above the level of the tooth roots and below the zygomatic body with or without repositioning of the mandible. The bones of the midface are sepa- rated from the cranial base, and the maxilla is advanced into the proper position. Preoperative planning should be undertaken with the orthodontist to determine the amount of advancement needed for proper occlusion and where the maxilla should be placed.
A splint is fashioned preoperatively so that intraoperatively, the maxilla can be advanced and placed into the splint. The maxilla is reaffixed to the cranium with plates and screws. In severe cases, if occlusion cannot be achieved with only the advancement of the maxilla, the mandible may also require operative adjustment to attain proper occlusion.
CHAPTER 32 / Plastic & Reconstructive Surgery 531

33
Q

7.
Ref: 1, 2, 24 Which of the following is true regarding nasal trauma?
A. A septal hematoma should not be disturbed and should be left to absorb on its own.
B. Diagnosis of nasal fractures requires a dedicated maxillofacial CT.
C. Naso-orbitoethmoid fractures are usually managed with closed reduction and splinting.
D. Patients with telecanthus usually require reattachment of the medial canthus tendons.
E. Nasolacrimal duct stenting is avoided.

532 SECTION VIII / Subspecialties for the General Surgeon

A

ANSWER: D
COMMENTS: A. False. When evaluating a patient with nasal trauma, the nasal septum should be examined for signs of a septal hematoma. If undiagnosed, a septal hematoma can lead to necrosis and erosion of the nasal septum and result in a saddle nose defor- mity. A septal hematoma should be incised and drained, and either quilting sutures with absorbable suture or a nasal stent should be placed to prevent reaccumulation.
B. False. Fractures of the nasal bones are diagnosed clinically, and dedicated radiography is rarely necessary. Nasal fractures are usually treated by closed reduction, splinting, and intranasal packing. If nasal packing is used, antibiotics should be initiated as cases of toxic shock have been described with intranasal packing.
C. False. Naso-orbitoethmoid fractures result from high- impact injuries and are associated with fractures of the nasal bones, ethmoid complex, medial orbital walls, nasofrontal junction, and nasal septum. The nasolacrimal duct may also be injured. Closed reduction is not sufficient to repair these injuries.
D. True. Telecanthus is the increased distance between the medial canthi of the eyes, and repair involves reattachment of the medial canthus tendons or repositioning of the bony segments to which the medial canthal tendons are attached.
E. False. Naso-orbitoethmoid fractures may involve disrup- tion of the nasolacrimal ducts, especially if telecanthus is present. These injuries should be repaired over a stent to avoid stricture.
Ref: 25

34
Q
  1. Which of the following statements regarding the facial nerve
    is true?
    A. The three major branches of the facial nerve are the
    ophthalmic, maxillary/buccal, and mandibular.
    B. Injury to the buccal branch of the facial nerve medial to the lateral canthus should be tagged or reapproximated within 72 h.
    C. A sural nerve graft may be used when a gap between the cut ends of the facial nerve precludes primary repair.
    D. Injury to the marginal mandibular branch of the facial nerve results in no significant functional sequelae.
    E. Gold upper eyelid implants are used sometimes in early reconstruction.
A

ANSWER: C
COMMENTS: A. False. The facial nerve, cranial nerve VII, has five major branches—the frontal/temporal, zygomatic, buccal, mar- ginal mandibular, and cervical. The frontal branch runs just deep to the superficial temporal fascia. The zygomatic, buccal, and mar- ginal mandibular branches run deep to the superficial musculoapo- neurotic system after traversing the substance of the parotid gland. The ophthalmic (V1), maxillary (V2), and mandibular (V3) branches are the three major branches of the trigeminal nerve, which supplies sensory innervation to the face.
B. False. Repair or identification plus tagging of facial nerve injuries is generally undertaken within the first 72 h following transec- tion. After transection, stimulation of the nerve branches with contrac- tion of the facial muscles occurs for up to 72 to 96 h after the injury, aiding in facial nerve branch identification. Generally, injuries to the zygomatic and buccal branches medial to the lateral canthus are not repaired because of difficulty in identifying these small branches and because of multiple interconnections with cross-innervation, which make the sacrifice of a peripheral branch insignificant most of the time.
C. True. Early reconstruction involves approximation of the cut nerve ends or reconstruction with a nerve graft (usually the sural nerve) to span a defect that cannot be approximated in a tension- free manner. For proximal facial nerve injuries, a cross-face nerve graft to the contralateral facial nerve may be undertaken.
D. False. Injury to peripheral branches of the frontal and marginal mandibular branches should be repaired whenever pos- sible because injury results in a significant functional loss. Bell palsy is the most common cause of facial paralysis, followed by facial trauma. Reconstruction for facial paralysis can be divided into early and late reconstruction.
E. False. About 18 months after the initial injury, the facial muscles atrophy and lose function. Once this happens, reconstruc- tion with muscle transfers is usually undertaken (e.g., gracilis muscle transfer with or without a cross-face nerve graft). Static reconstructions with gold upper eyelid implants, browlifts, and blepharoplasty are also options for delayed reconstruction.
Ref: 1, 3

35
Q
  1. Which of the following statements regarding frontal sinus
    fractures is true?
    A. Frontal sinus fractures usually occur in isolation as a
    result of blast injuries.
    B. Nondisplaced fractures of the anterior wall of the frontal sinus require operative exploration, mucosal stripping of the sinus, and rigid fixation.
    C. Treatment of frontal sinus fractures with significant disruption of the posterior wall and associated cerebrospi- nal fluid (CSF) leak involves cranialization of the sinus.
    D. Anterior wall fractures involving the nasofrontal duct require stenting the nasofrontal duct with a Silastic stent for 2 weeks.
    E. Cranialization involves decompressing the sinus into the peri-cavernous lymphatics.
A

ANSWER: C
COMMENTS: A. False. Frontal sinus fractures are usually the result of a high-energy impact. The majority of frontal sinus frac- tures are associated with other maxillofacial and intracranial inju- ries. CT is generally used to diagnose frontal sinus fractures. All patients suspected of having a frontal sinus injury should be evalu- ated for signs of CSF rhinorrhea, which indicates a dural tear. This will clinically present as clear persistent nasal drainage or salty discharge.
B. False. Some advocate the observation of nondisplaced or isolated posterior wall fractures without evidence of CSF leak, whereas others advocate exploration of all posterior wall fractures. Antibiotics should be considered in patients with suspected CSF rhinorrhea. Nondisplaced anterior table fractures without posterior table involvement typically do not require operative intervention.
C. True. Treatment of a frontal sinus fracture depends on the number of walls involved, the status of the nasofrontal duct, and the degree of displacement.
D. False. Isolated fractures of the anterior wall are treated only if there is considerable displacement or esthetic defect. If the nasofron- tal duct is involved in the fracture, the frontal sinus is demucosalized, the nasofrontal duct is plugged with a bone graft or other material, and the sinus is obliterated with cancellous bone or fat. Fractures of the posterior wall raise suspicion for a dural laceration.
E. False. For displaced fractures of the posterior wall, a team approach with a neurosurgeon is preferred. If there is a significant

loss of the posterior wall, the sinus is cranialized. Cranialization involves removing the posterior wall, plugging the nasofrontal duct with bone graft or other material, demucosalizing the sinus, and replacing the sinus with a pericranial flap.
Ref: 1, 25

36
Q
  1. Which of the following is true regarding the repair of orbital
    floor fractures?
    A. Cosmetically unacceptable enophthalmos is not an indication for surgical repair.
    B. Diplopia is an indication for surgical exploration.
    C. Exposure of the orbital floor via a subciliary incision is associated with less risk for lower lid retraction than is exposure via a transconjunctival incision.
    D. Marked periorbital swelling is an indication for urgent surgical exploration.
    E. Treatment of complex bone fractures take precedence over that of globe injuries.
A

ANSWER: B
COMMENTS: The diagnosis of an orbital floor fracture is sus- pected when periorbital ecchymosis and subconjunctival hematoma are present. Patients may also exhibit anesthesia in the sensory distribution of the infraorbital nerve, which lies beneath the orbital floor. Orbital floor fractures may also be manifested as diplopia and enophthalmos. Diplopia may result from the restriction of extraocu- lar movement because of contusion or entrapment of the inferior rectus or inferior oblique muscles in the fracture segment. Enoph- thalmos, or posterior displacement of the globe, results from increased orbital volume (as the floor is displaced inferiorly) or is
caused by disruption of the ligamentous support of the globe. Initial evaluation of a patient with a suspected orbital fracture includes testing for visual acuity, extraocular muscle movement, and pupil- lary reflexes. A forced duction test whereby the insertion of the inferior rectus is grasped with forceps and manually rotated (after a topical anesthetic is instilled) helps identify extraocular muscle entrapment. If visual acuity is affected or globe rupture is sus- pected, ophthalmologic consultation is mandatory.
A. False. Cosmetically unacceptable enophthalmos is one of the indications for surgical repair.
B. True. The indications for surgical repair of orbital floor fractures include diplopia, entrapment of extraocular muscles, and enophthalmos. Surgery may be performed immediately or delayed until the edema has resolved, usually within a 2- to 3-week period from the time of injury. Exposure is usually achieved with either a transconjunctival incision or subciliary or subtarsal incisions. Sub- ciliary and subtarsal incisions carry a greater risk for lower lid retraction or ectropion. Many materials have been used to recon- struct the orbital floor, including titanium mesh, polyethylene sheets, and bioresorbable mesh.
C. False. Exposure of the orbital floor via a subciliary incision is associated with a higher risk for lower lid retraction than is exposure via a transconjunctival incision.
D. False. Marked periorbital swelling is not an indication for urgent surgical exploration.
E. False. Globe injury always takes precedence over bone repair. Fine-cut maxillofacial CT is performed with both axial images and coronal reconstructions to better characterize the nature of the fracture and aid in the identification of an entrapped extra- ocular muscle.
Ref: 1, 25
CHAPTER 32 / Plastic & Reconstructive Surgery 533

534 SECTION VIII / Subspecialties for the General Surgeon G. Hand

37
Q
  1. An 8-year-old child is taken to the emergency department after slamming his right index finger in a door. There is a laceration on the pulp of the finger and a subungual hema- toma. A radiograph shows a tuft fracture. Your treatment plan should include which of the following:
    A. Leave the laceration open, dress the finger with antibiotic ointment, and place it in a finger splint.
    B. Perform a finger block, remove the nail plate, repair any nail bed laceration, and repair the pulp laceration.
    C. Use Kirschner wire fixation of the fracture, remove the nail plate, and repair the nail bed.
    D. Perform operative exploration with internal fixation of the tuft fracture and repair the nail bed and pulp lacerations.
    E. Place the eponychial fold over the nail plate.
A

ANSWER: B
COMMENTS: A., C., D., and E. Incorrect. These choices do not describe the correct treatment plan. Distal phalangeal fractures are among the most common fractures seen in the hand, and of these, tuft fractures are the most common. These fractures are often com- minuted and are frequently associated with a nail bed injury.
B. Correct. Treatment involves removal of the nail plate, irrigation, repair of the nail bed with absorbable suture, replace- ment of the nail plate under the eponychial fold to prevent scar- ring or synechiae formation of the fold to the nail plate, and use of a hand or finger splint. If the nail plate is not available, an alternative material can be used to splint the proximal eponychial fold. Such management allows a new nail to grow out from under the eponychial fold. Most of these fractures can be reduced at the time of nail bed repair and protected with a splint for 3 to 4 weeks. Kirschner wire fixation and operative repair are rarely indicated.
Ref: 1, 5

38
Q
  1. Which of the following statements regarding the placement
    of hand incisions is true?
    A. Palm incisions should be placed in the skin creases.
    B. It is better to err on the volar aspect than on the dorsal aspect when placing incisions on the side of the digit.
    C. Incisions on the volar side of the digit must cross the interphalangeal (IP) flexion creases transversely.
    D. Dorsal skin incisions should cross skin creases trans- versely or obliquely.
    E. The key principle in planning hand incisions is to maximize motion to avoid contractures.
A

ANSWER: D
COMMENTS: A. and C. False. There are several key principles in planning hand incisions. Oblique incisions connecting these points (Bruner), or volar zigzag incisions, are also an excellent approach that provides full exposure to the entire palmar side of
the digit. A line marking the change in character between the dorsal and volar skin of the digit is also a useful landmark.
Those on the palm should run parallel to the skin creases or across them obliquely because the blood supply in this region comes straight upward into the skin. Digital incisions should be placed dorsal to the midlateral line through the midaxial line, which is exactly neutral between flexion and extension. This line is deter- mined by connecting the most dorsal points of the IP joint creases when the finger is in a flexed position.
B. False. Given a choice, it is far better to err in placing an incision dorsally than on the volar aspect of the side of the digit because the volar incisions may form a bridging scar.
D. True. Skin incisions on the dorsum of the hand and digits should cross skin creases transversely, obliquely, or over the mid- dorsum when between joints. In a rheumatoid hand, incisions that cross the skin of the dorsal surface of the wrist should be longitu- dinal or minimally curved to avoid slough of a distally based flap.
E. False. Whenever possible, the incision should be designed along lines that undergo no change in length with motion.
Ref: 1, 5, 27

39
Q
  1. A surgeon is called to examine a 30-year-old painter who cut the palm of his right hand with a dirty razor blade. Examina- tion reveals a 2-cm clean laceration at the base of the long finger. Metacarpophalangeal (MCP) joint flexion is intact, but the patient cannot flex either IP joint in that finger. The injury is 1 h old. What is the diagnosis?
    A. Lacerated flexor digitorum superficialis (FDS) tendon B. Lacerated flexor digitorum profundus (FDP) tendon C. Combined FDS and FDP laceration
    D. Laceration of the intrinsic muscles to the long finger E. Median nerve transection
A

ANSWER: C
COMMENTS: A. Incorrect. Lacerated FDS tendon would not affect the ability to flex at the distal interphalangeal (DIP) joint.
B. Incorrect. Lacerated FDP tendon would not affect the ability to flex at the proximal interphalangeal (PIP) joint.
C. Correct. The patient’s laceration at the base of the long finger is within zone II, where the FDS and FDP are within the same tendon sheath and vulnerable to simultaneous injury. The patient has also lost flexion at both the DIP and PIP joints, demon- strating an injury to both the FDS and FDP tendons.
D. Incorrect. This patient is able to flex the MCP of his injured finger, demonstrating that the lumbricals are intact. Injury to other intrinsic muscles of the hand would not result in the loss of IP joint flexion.
E. Incorrect. Transection of the median nerve at the hand may result in numbness of the thumb and first two fingers or the loss of thumb adduction and opposition. However, a distal median nerve injury would not result in loss of the IP joints as is seen with a proximal median nerve injury.
Ref: 1, 5, 27

40
Q
  1. The immediate treatment plan for the patient described in Question 345 should include which of the following?
    A. Plans for immediate tendon repair (within 6 h) to avoid the hazards of delayed tendon anastomosis
    B. Wrist block anesthesia, extension of the skin wound along proper incision lines, and exploration to confirm the diagnosis
    C. Careful cleansing and irrigation of the wound, placement of an appropriate dressing or simple sutures, and hand immobilization before definitive primary surgical repair within 2 weeks
    D. Cleansing of the wound, primary skin closure, hand immobilization, and outpatient follow-up visits because this injury will require free tendon graft reconstruction 6 weeks after injury
    E. Cleansing of the wound, closure, and then physical therapy starting 2 weeks after the injury
A

ANSWER: C
COMMENTS: A., B., D., and E. Incorrect. These choices do not describe the correct treatment plan. Flexion of the MCP joint is a func- tion of the intrinsic muscles and can persist in the face of extrinsic flexor muscle and tendon injury. The goal of flexor tendon repair is the restoration of IP joint flexion. Flexor tendon repair demands meticu- lous attention to detail and, whenever possible, should be performed by a hand surgeon. The character of the wound, the nature of the injury, the degree of contamination, and the time between injury and definitive treatment determine whether primary or delayed repair is performed.
C. Correct. Proper wound cleansing, dressing, immobiliza- tion, and prophylactic antibiotics allow delay of primary repair if a hand surgeon is not immediately available. The hand should be immobilized in a neutral or slightly flexed position. If there is a question about the degree of contamination or if the initial wound treatment has been delayed beyond several hours thus making primary closure hazardous, delayed repair after 2 to 14 days may be performed. This allows the presence or absence of infection to be clearly established. Many experts believe that this type of delay does not significantly alter the ultimate outcome of the repair.
Tendon injuries with grossly contaminated wounds, those with significant tendon loss, or wounds with significant associated injuries to the soft tissue, bone, nerve, or blood vessels should be treated by secondary repair in 3 to 6 weeks—after the wounds have stabilized, the infection has cleared, and edema formation has subsided.
Ref: 1, 5, 27

41
Q
  1. Which of the following statements regarding the most
    common metacarpal fractures is true?
    A. They are commonly known as Bennett fractures.
    B. They are commonly known as boxer’s fractures.
    C. They most often involve the distal metacarpal of the index and long fingers.
    D. Physical examination is the most effective means of assessing the degree of angulation.
    E. They usually require open reduction and internal fixation.
A

ANSWER: B
COMMENTS: A. False. A Bennett fracture is a fracture at the base of the thumb metacarpal and not the most common metacarpal fracture.
B. True. Metacarpal fractures commonly result from hitting an object with a clenched fist. They usually involve the distal metacarpal of the fifth and occasionally the fourth fingers and are known as “boxer’s fractures.” The metacarpal head is displaced palmward, and pain, swelling, and some loss of knuckle promi- nence are the usual physical findings. Associated lacerations should be treated as human bites until proved otherwise, and early explora- tion, with the administration of intravenous antibiotics, is recommended.
D. False. Physical examination would not be sufficient to evaluate the injury because swelling usually masks the degree of angulation. A lateral radiograph is needed for accurate evaluation. Each finger should be individually flexed to the palm to assess the degree of rotational deformity. During flexion, the fingers normally point to the scaphoid tubercle. Deviation from this alignment allows estimation of the rotational deformity.
E. False. The usual treatment is closed reduction followed by immobilization of the involved and adjacent digits, with the MCP joint placed in 65 to 90 degrees of flexion and the IP joints placed in full extension in the intrinsic plus position.
Unstable or multiple metacarpal fractures often require open reduction and internal fixation. Metacarpal shaft fractures require reduction and immobilization. Percutaneous Kirschner wires or a plate and screws for internal fixation are frequently required if the fracture is unstable, particularly if the fracture is oblique or comminuted.
Ref: 1, 5, 27, 28

42
Q
  1. Which of the following statements regarding tenosynovitis is
    true?
    A. Infections of the flexor sheath of the little finger more
    often extend to the thumb than to the adjacent ring finger.
    B. A flexor tendon sheath infection causes the involved finger to assume a position of mild extension at all joints.
    C. The involved digit is rarely swollen and often exhibits little pain.
    D. By definition, deep palmar space infections involve the flexor tendons.
    E. Because of potential for contracture, conservative management without drainage is contraindicated.
A

ANSWER: A
COMMENTS: A. True. The flexor sheath of the little finger com- municates with the ulnar bursa. The flexor sheath of flexor pollicis longus communicates with the radial bursa. An infection in the ulnar bursa and radial bursa at the level of the wrist can lead to a “horseshoe” abscess involving both radial and ulnar bursae.
B. and C. False. The cardinal signs of suppurative flexor tenosynovitis are known as Kanavel’s signs. These include fusi- form swelling of the digit, digit held in mid-flexed position, severe pain with passive extension of the digit, and tenderness along the entire flexor tendon sheath. Pain with passive extension of the digit is considered the most sensitive sign for flexor tenosynovitis.
D. False. In the palm, the deep spaces are divided into the thenar space and the midpalmar space at the level of the third metacarpal, where a vertical septum extends between the metacar- pal and sheath of the long finger flexor tendons. Infection here is manifested as localized, tender swelling and must be drained with an appropriate incision.
E. False. Infection of the synovial sheaths of the flexor tendons is a serious problem that requires prompt appropriate treatment
CHAPTER 32 / Plastic & Reconstructive Surgery 535

536 SECTION VIII / Subspecialties for the General Surgeon
including proper drainage and antibiotics. The tendons are rela- tively avascular and are characterized by poor natural resistance to infection. This may be compounded in patients with diabetes mel- litus or in those who are immunocompromised.
Ref: 1, 5 27, 28

43
Q
  1. Which of the following statements regarding replantation of
    the hand is false?
    A. Single digits (other than the thumb) are uncommonly
    replanted except in children.
    B. The amputated part may tolerate cool ischemia for up to 24 h if there is no significant avascular muscle mass.
    C. Bleeding from the proximal part is ideally treated with pressure rather than with clamping.
    D. A history of heavy smoking, diabetes mellitus, hyperten- sion, and Raynaud phenomenon are relative contraindica- tions to replantation.
    E. Replantation above the elbow is contraindicated.
A

ANSWER: E
COMMENTS: A. True. Replantation is a highly specialized pro- cedure that is best performed by a team of replantation surgeons. Single digits are less frequently replanted, except in children or if there is a sharp noncrushing cut at the level of the middle phalanx distal to the splitting of the superficialis tendon.
The hand and thumb are always considered for replantation unless definite contraindications exist or the extremity was not properly preserved. The thumb is the most important digit, and as much of its length as possible should be preserved. An index finger amputated proximal to the PIP joint loses its ability to pinch, and the brain naturally switches the pinching to the long finger. Attempts to preserve length distal to the PIP joint should be made, but if the digit is painful, is insensate, or “gets in the way,” the patient may best be served by transection through the metacarpal (ray amputa- tion) because the long finger can assume the role of primary pinch. Loss of little finger length proximal to the PIP joint may also best be treated by ray amputation.
B. True. Distal amputations properly cooled immediately after injury may be viable for up to 24 h. Central finger amputations (long and ring fingers) near the MCP joint can cause bothersome spaces in the clenched fist that can be treated by transfer of the adjacent peripheral finger with its metacarpal to fill the space.
C. True. Bleeding should be controlled with pressure. Clamp- ing or ligating the bleeding vessels should not be attempted.
D. True. Relative contraindications to replantation include medically unstable patients (i.e., history of smoking, diabetes mel- litus, and hypertension), prolonged ischemia time with avascular muscle mass, and tissue contamination.
E. False. Amputations above the elbow are considered for replantation (particularly in children) because even partial success can convert an above-elbow to a below-elbow stump for future rehabilitation. Guillotine amputations are the injuries that are most favorable for replantation.
Ref: 1, 5, 27

44
Q
  1. Which of the following statements regarding peripheral nerve
    injury is false?
    A. Nerve repair is progressively less effective if delayed
    beyond 2 months after surgery.
    B. Nerve repair is best performed under 4 to 15 times magnification.
    C. Nerve repair is best performed in a fashion that mini- mizes tension across the repair.
    D. Regeneration may be followed clinically by observing the distal progression of the Tinel sign.
    E. The use of conduits can assist in nerve regeneration.
A

ANSWER: A
COMMENTS: A. False. Nerve injuries do not need to be repaired at the time of injury, but it is believed that the results are progres- sively worse if the repair is delayed beyond 6 months, when the distal nerve tubules have contracted and the new axons can no longer grow distally. Occasionally, repair delayed for up to 2 years has been successful, particularly in children.
B. and C. True. Nerve injuries result from stretching or com- pression (neurapraxia) or from transection. Neurapraxic injuries carry a better prognosis than do transection injuries. Most surgeons perform a careful epineural repair at 4 to 15 times magnification with minimal tension. However, repair of the individual fascicular bundle may be required in large, mixed peripheral nerves. Recovery after repair begins with the return of function starting proximally. Regenerating axons grow down the distal nerve sheath at the rate of approximately 1 mm/day.
D. True. Distal progression of the Tinel sign (tingling felt after percussion over the growing nerve) usually follows the start of regeneration.
E. True. Recently, successful nerve regeneration with healing has been achieved with vein grafts, synthetic tubes, and cadaver allograft, which act as conduits for nerve growth across the repair.
Ref: 1, 27, 28

45
Q
  1. A 70-year-old mechanic sustains a “pinching” amputation when his dominant long finger is caught between a garage door pulley and the belt. There is loss of the volar two-thirds of the pulp skin, along with exposed subcutaneous tissue. Select the most appropriate reconstruction:
    A. Sterile dressing changes with topical antibiotics and closure by contracture and epithelialization
    B. Full-thickness hypothenar skin graft
    C. STSG
    D. V-Y advancement flap
    E. Replantation
A

ANSWER: B
COMMENTS: A. Incorrect. Sterile dressing changes with topical antibiotics and closure by contracture and epithelialization would not be appropriate to reconstruct this defect.
B. Correct. In this particular case, a hypothenar skin graft is best. It provides glabrous skin (the unique non–hair-bearing skin found on the volar aspect of the digits and palms, as well as the soles), which matches the other digits and resists contracture and hypersensitivity. Sensibility, an important factor with all hand grafts, is similar to that achieved with flaps.
Fingertip amputation is one of the most common hand injuries. The mechanism of injury; the orientation and location of the ampu- tation; and the age, gender, general condition, and hand dominance of the patient are integral to decision making and planning of the reconstruction. Reconstructions that require prolonged immobiliza- tion in “unsafe” positions, such as a thenar flap or a cross-finger flap, are not recommended in older individuals due to the risk of immobilization. If properly cared for and free of crush-avulsion

injury, the amputated part can be defatted, meticulously sutured back, and used for a salvage procedure.
C. Incorrect. An STSG would be too thin, and allowing the wound to heal by contracture and epithelialization is a good option when there is only soft tissue loss, but the orientation should be such that the tissues can contract to cover the defect. The broad surface area described here would not be satisfactory for this method of healing.
D. Incorrect. The V-Y advancement flap is best used with straight transverse amputations.
E. Incorrect. Microvascular replantations have been success- ful even as far distal as the mid-nail but are not generally performed for fingertip injuries.

46
Q
  1. Which of the following statements is false? joint.
    B. Mallet finger results from traumatic avulsion of the extensor tendon insertion into the distal phalanx.
    C. Skier’s (ski pole injury) thumb results from disruption of the ulnar collateral ligament at the thumb MCP joint.
    D. Kienböck disease refers to idiopathic lunatomalacia.
    E. Rolando and Bennett fractures are fractures of the base of
    the thumb metacarpal.
A

ANSWER: A
COMMENTS: A. False. There are numerous common names for disorders of the hand. Understanding the cause of the disorder sheds light on the common name. Jersey finger refers to a traumatic avulsion of the FDP at the DIP level. It is an injury sustained most commonly by rugby and football players as the tendon avulses from the bone with forceful gripping, such as when grabbing a jersey most commonly affecting the ring digit.
B. True. Mallet finger is the extensor counterpart of jersey finger and is a traumatic disruption of the extensor mechanism at its insertion into the distal phalanx that is usually caused by forced flexion at the joint, such as the one that occurs when “jamming” one’s long finger while catching a softball. Injuries to the extensor insertion into the dorsum of the distal phalanx result in the mallet deformity. Frequently, no fracture can be seen.
If such an injury is associated with no fracture or is associated with a fracture and the fragment is small, dorsal splinting with the joint in 0 to 10 degrees of hyperextension for 6 to 8 weeks provides good results. If more than one-third of the articular surface is dis- placed with the avulsed tendon and volar subluxation of the distal phalanx occurs, open reduction plus internal fixation is advised.
A boutonnière deformity results from disruption of the central extensor tendon at its insertion into the dorsum of the base of the middle phalanx. Immobilization of the PIP joint at zero degrees of extension with dynamic extension splinting for 6 to 8 weeks is recommended whenever possible. The name gamekeeper’s thumb is derived from Scottish gamekeepers’ practice of breaking the necks of rabbits by twisting their necks. The repetitive trauma of this type tended to weaken the ulnar collateral ligament of the thumb and cause both pain and laxity of the thumb at the MCP joint in this area.
C. True. Skier’s (ski pole injury) thumb is specifically an acute traumatic avulsion of the ulnar collateral ligament, although game- keeper’s thumb is sometimes used incorrectly to describe this situation.
D. True. Robert Kienböck, an Austrian professor of radiology, was the first to describe a syndrome of idiopathic lunatomalacia in 1910—Kienböck disease. It is most commonly manifested as a stiff, painful, and weak wrist in a young adult male.
E. True. A Bennett fracture is a fracture at the base of the thumb metacarpal. It is inherently unstable, but its management tends to be less complicated and its prognosis much better than that of a Rolando fracture. The latter is a comminuted, intraarticular fracture at the base of the thumb metacarpal that exhibits a T or Y fracture pattern in which the articular surface is split at the base of the first metacarpal.